Showing a polynomial having at least one integer root under certain conditions has precisely one integer root...












9














$P(x) = 0$ is a polynomial equation having at least one integer root, where $P(x)$ is a polynomial of degree five and having integer coefficients. If $P(2) = 3$ and $P(10)= 11$, then prove that the equation $P(x) = 0$ has exactly one integer root.



I tried by assuming a fifth degree polynomial but got stuck after that.



The question was asked by my friend.










share|cite|improve this question















put on hold as off-topic by Saad, Xander Henderson, Did, user91500, Paul Frost 21 hours ago


This question appears to be off-topic. The users who voted to close gave this specific reason:


  • "This question is missing context or other details: Please provide additional context, which ideally explains why the question is relevant to you and our community. Some forms of context include: background and motivation, relevant definitions, source, possible strategies, your current progress, why the question is interesting or important, etc." – Saad, Xander Henderson, Did, user91500, Paul Frost

If this question can be reworded to fit the rules in the help center, please edit the question.









  • 4




    "The question was asked by my friend." Sorry but why are you mentioning this? Is this suposed to make any difference (for example, say, to alleviate the lack of personal input)?
    – Did
    Jan 1 at 11:49
















9














$P(x) = 0$ is a polynomial equation having at least one integer root, where $P(x)$ is a polynomial of degree five and having integer coefficients. If $P(2) = 3$ and $P(10)= 11$, then prove that the equation $P(x) = 0$ has exactly one integer root.



I tried by assuming a fifth degree polynomial but got stuck after that.



The question was asked by my friend.










share|cite|improve this question















put on hold as off-topic by Saad, Xander Henderson, Did, user91500, Paul Frost 21 hours ago


This question appears to be off-topic. The users who voted to close gave this specific reason:


  • "This question is missing context or other details: Please provide additional context, which ideally explains why the question is relevant to you and our community. Some forms of context include: background and motivation, relevant definitions, source, possible strategies, your current progress, why the question is interesting or important, etc." – Saad, Xander Henderson, Did, user91500, Paul Frost

If this question can be reworded to fit the rules in the help center, please edit the question.









  • 4




    "The question was asked by my friend." Sorry but why are you mentioning this? Is this suposed to make any difference (for example, say, to alleviate the lack of personal input)?
    – Did
    Jan 1 at 11:49














9












9








9


7





$P(x) = 0$ is a polynomial equation having at least one integer root, where $P(x)$ is a polynomial of degree five and having integer coefficients. If $P(2) = 3$ and $P(10)= 11$, then prove that the equation $P(x) = 0$ has exactly one integer root.



I tried by assuming a fifth degree polynomial but got stuck after that.



The question was asked by my friend.










share|cite|improve this question















$P(x) = 0$ is a polynomial equation having at least one integer root, where $P(x)$ is a polynomial of degree five and having integer coefficients. If $P(2) = 3$ and $P(10)= 11$, then prove that the equation $P(x) = 0$ has exactly one integer root.



I tried by assuming a fifth degree polynomial but got stuck after that.



The question was asked by my friend.







polynomials diophantine-equations






share|cite|improve this question















share|cite|improve this question













share|cite|improve this question




share|cite|improve this question








edited Dec 31 '18 at 6:54









hardmath

28.7k95095




28.7k95095










asked Dec 30 '18 at 8:34









Ramanujam Ganit Prashikshan Ke

16919




16919




put on hold as off-topic by Saad, Xander Henderson, Did, user91500, Paul Frost 21 hours ago


This question appears to be off-topic. The users who voted to close gave this specific reason:


  • "This question is missing context or other details: Please provide additional context, which ideally explains why the question is relevant to you and our community. Some forms of context include: background and motivation, relevant definitions, source, possible strategies, your current progress, why the question is interesting or important, etc." – Saad, Xander Henderson, Did, user91500, Paul Frost

If this question can be reworded to fit the rules in the help center, please edit the question.




put on hold as off-topic by Saad, Xander Henderson, Did, user91500, Paul Frost 21 hours ago


This question appears to be off-topic. The users who voted to close gave this specific reason:


  • "This question is missing context or other details: Please provide additional context, which ideally explains why the question is relevant to you and our community. Some forms of context include: background and motivation, relevant definitions, source, possible strategies, your current progress, why the question is interesting or important, etc." – Saad, Xander Henderson, Did, user91500, Paul Frost

If this question can be reworded to fit the rules in the help center, please edit the question.








  • 4




    "The question was asked by my friend." Sorry but why are you mentioning this? Is this suposed to make any difference (for example, say, to alleviate the lack of personal input)?
    – Did
    Jan 1 at 11:49














  • 4




    "The question was asked by my friend." Sorry but why are you mentioning this? Is this suposed to make any difference (for example, say, to alleviate the lack of personal input)?
    – Did
    Jan 1 at 11:49








4




4




"The question was asked by my friend." Sorry but why are you mentioning this? Is this suposed to make any difference (for example, say, to alleviate the lack of personal input)?
– Did
Jan 1 at 11:49




"The question was asked by my friend." Sorry but why are you mentioning this? Is this suposed to make any difference (for example, say, to alleviate the lack of personal input)?
– Did
Jan 1 at 11:49










3 Answers
3






active

oldest

votes


















25














The assumption that $P$ has degree $5$ is irrelevant and unhelpful.



If $r$ is a root of $P$, we can write $P(x)=(x-r)Q(x)$ for some polynomial $Q$. If $r$ is an integer, then $Q$ will also have integer coefficients (polynomial division never requires dividing coefficients if you are dividing by a monic polynomial). So, for any integer $a$, $P(a)=(a-r)Q(a)$ must be divisible by $a-r$. Taking $a=2$ and $a=10$, we see easily that the only possible value of $r$ is $-1$.



Moreover, we can say that $P$ only has one integer root even counting multiplicity, because if $-1$ were a root of higher multiplicity, we could write $P(x)=(x+1)^2R(x)$ where $R(x)$ again has integer coefficients, so $P(2)$ would need to be divisible by $(2+1)^2=9$.






share|cite|improve this answer





























    28














    If $u$ and $v$ are integer roots of $P$, then $P(x)=(x-u)(x-v)Q(x)$, where $Q$ is a polynomial with integer coefficients. From $P(2)=3$ we get $(u-2)(v-2)mid 3$, and then WLOG either $u-2=1$ or $u-2=-1$, implying $uin{1,3}$. Now $P(10)=11$ gives $(u-10)(v-10)mid 11$, showing that $u-10$ is a divisor of $11$. However, neither $1-10=-9$, not $3-10=-7$ is a divisor of $11$, a contradiction.






    share|cite|improve this answer



















    • 1




      No idea why your answer has gotten so many fewer votes than mine! Your approach of considering both factors at once is quite slick.
      – Eric Wofsey
      Dec 30 '18 at 16:54






    • 2




      @EricWofsey: Don't worry, that's fine. Your answer provides some theoretical background and looks "more scientific". Besides, my empirical observation is that the number of upvotes is proportional to the present reputation of the person answering :-)
      – W-t-P
      Dec 30 '18 at 17:19



















    1














    Key Idea $ $ (Kronecker) $ $ How polynomials can factor is constrained by how their values factor, $ $ e.g. as below, in some cases if $,P,$ takes a prime value then it has at most one integer root.



    Hint $ $ If $,P,$ has more roots than $,P(2),$ has prime factors then factoring $P$ & evaluating at $x!=!2$ $,Rightarrow,P(1)!=!0,$ or $P(3)!=!0.,$ But $P(1)!neq! 0,$ else $,10!-!1mid P(10)!-!P(1) = 11.,$ $P(3)!neq! 0,$ similarly.



    Theorem $ $ Suppose $P(x)$ is a polynomial with integer coefficients and $a$ is an integer with $,P(a)neq 0,$ and there exists an integer $b$ such that neither of $,b!-!apm 1$ divides $P(b).$



    $$begin{align} {rm Then} &P(a), text{has $, k, $ prime factors (counting multiplicity)}\
    Longrightarrow &P(x), text{ has $le! k,$ integer roots (counting multiplicty)}
    end{align}qquad $$



    Proof $ $ If not then $P$ has at least $,k+1,$ roots $,r_i,$ so iterating the Factor Theorem yields
    $$,P(x) = (x-r_0)cdots (x-r_k),q(x)qquad$$



    for a polynomial $,q(x),$ with integer coefficients. Evaluating above at $,x = a,$ yields



    $$,P(a) = (a-r_0)cdots (a-r_k),q(a)qquad$$



    If all $,a-r_ineq pm1,$ then they all have a prime factor yielding at least $k+1$ prime factors on the RHS, contra LHS $,P(a),$ has $,k,$ prime factors (prime factorizations are unique). So some $,a-r_j = pm1,$ so $,r_j = apm 1.,$ Evaluating at $, x = b,$ yields $,b-r_j = b-apm1,$ divides $, P(b),,$ contra hypothesis.






    share|cite|improve this answer






























      3 Answers
      3






      active

      oldest

      votes








      3 Answers
      3






      active

      oldest

      votes









      active

      oldest

      votes






      active

      oldest

      votes









      25














      The assumption that $P$ has degree $5$ is irrelevant and unhelpful.



      If $r$ is a root of $P$, we can write $P(x)=(x-r)Q(x)$ for some polynomial $Q$. If $r$ is an integer, then $Q$ will also have integer coefficients (polynomial division never requires dividing coefficients if you are dividing by a monic polynomial). So, for any integer $a$, $P(a)=(a-r)Q(a)$ must be divisible by $a-r$. Taking $a=2$ and $a=10$, we see easily that the only possible value of $r$ is $-1$.



      Moreover, we can say that $P$ only has one integer root even counting multiplicity, because if $-1$ were a root of higher multiplicity, we could write $P(x)=(x+1)^2R(x)$ where $R(x)$ again has integer coefficients, so $P(2)$ would need to be divisible by $(2+1)^2=9$.






      share|cite|improve this answer


























        25














        The assumption that $P$ has degree $5$ is irrelevant and unhelpful.



        If $r$ is a root of $P$, we can write $P(x)=(x-r)Q(x)$ for some polynomial $Q$. If $r$ is an integer, then $Q$ will also have integer coefficients (polynomial division never requires dividing coefficients if you are dividing by a monic polynomial). So, for any integer $a$, $P(a)=(a-r)Q(a)$ must be divisible by $a-r$. Taking $a=2$ and $a=10$, we see easily that the only possible value of $r$ is $-1$.



        Moreover, we can say that $P$ only has one integer root even counting multiplicity, because if $-1$ were a root of higher multiplicity, we could write $P(x)=(x+1)^2R(x)$ where $R(x)$ again has integer coefficients, so $P(2)$ would need to be divisible by $(2+1)^2=9$.






        share|cite|improve this answer
























          25












          25








          25






          The assumption that $P$ has degree $5$ is irrelevant and unhelpful.



          If $r$ is a root of $P$, we can write $P(x)=(x-r)Q(x)$ for some polynomial $Q$. If $r$ is an integer, then $Q$ will also have integer coefficients (polynomial division never requires dividing coefficients if you are dividing by a monic polynomial). So, for any integer $a$, $P(a)=(a-r)Q(a)$ must be divisible by $a-r$. Taking $a=2$ and $a=10$, we see easily that the only possible value of $r$ is $-1$.



          Moreover, we can say that $P$ only has one integer root even counting multiplicity, because if $-1$ were a root of higher multiplicity, we could write $P(x)=(x+1)^2R(x)$ where $R(x)$ again has integer coefficients, so $P(2)$ would need to be divisible by $(2+1)^2=9$.






          share|cite|improve this answer












          The assumption that $P$ has degree $5$ is irrelevant and unhelpful.



          If $r$ is a root of $P$, we can write $P(x)=(x-r)Q(x)$ for some polynomial $Q$. If $r$ is an integer, then $Q$ will also have integer coefficients (polynomial division never requires dividing coefficients if you are dividing by a monic polynomial). So, for any integer $a$, $P(a)=(a-r)Q(a)$ must be divisible by $a-r$. Taking $a=2$ and $a=10$, we see easily that the only possible value of $r$ is $-1$.



          Moreover, we can say that $P$ only has one integer root even counting multiplicity, because if $-1$ were a root of higher multiplicity, we could write $P(x)=(x+1)^2R(x)$ where $R(x)$ again has integer coefficients, so $P(2)$ would need to be divisible by $(2+1)^2=9$.







          share|cite|improve this answer












          share|cite|improve this answer



          share|cite|improve this answer










          answered Dec 30 '18 at 9:14









          Eric Wofsey

          180k12207335




          180k12207335























              28














              If $u$ and $v$ are integer roots of $P$, then $P(x)=(x-u)(x-v)Q(x)$, where $Q$ is a polynomial with integer coefficients. From $P(2)=3$ we get $(u-2)(v-2)mid 3$, and then WLOG either $u-2=1$ or $u-2=-1$, implying $uin{1,3}$. Now $P(10)=11$ gives $(u-10)(v-10)mid 11$, showing that $u-10$ is a divisor of $11$. However, neither $1-10=-9$, not $3-10=-7$ is a divisor of $11$, a contradiction.






              share|cite|improve this answer



















              • 1




                No idea why your answer has gotten so many fewer votes than mine! Your approach of considering both factors at once is quite slick.
                – Eric Wofsey
                Dec 30 '18 at 16:54






              • 2




                @EricWofsey: Don't worry, that's fine. Your answer provides some theoretical background and looks "more scientific". Besides, my empirical observation is that the number of upvotes is proportional to the present reputation of the person answering :-)
                – W-t-P
                Dec 30 '18 at 17:19
















              28














              If $u$ and $v$ are integer roots of $P$, then $P(x)=(x-u)(x-v)Q(x)$, where $Q$ is a polynomial with integer coefficients. From $P(2)=3$ we get $(u-2)(v-2)mid 3$, and then WLOG either $u-2=1$ or $u-2=-1$, implying $uin{1,3}$. Now $P(10)=11$ gives $(u-10)(v-10)mid 11$, showing that $u-10$ is a divisor of $11$. However, neither $1-10=-9$, not $3-10=-7$ is a divisor of $11$, a contradiction.






              share|cite|improve this answer



















              • 1




                No idea why your answer has gotten so many fewer votes than mine! Your approach of considering both factors at once is quite slick.
                – Eric Wofsey
                Dec 30 '18 at 16:54






              • 2




                @EricWofsey: Don't worry, that's fine. Your answer provides some theoretical background and looks "more scientific". Besides, my empirical observation is that the number of upvotes is proportional to the present reputation of the person answering :-)
                – W-t-P
                Dec 30 '18 at 17:19














              28












              28








              28






              If $u$ and $v$ are integer roots of $P$, then $P(x)=(x-u)(x-v)Q(x)$, where $Q$ is a polynomial with integer coefficients. From $P(2)=3$ we get $(u-2)(v-2)mid 3$, and then WLOG either $u-2=1$ or $u-2=-1$, implying $uin{1,3}$. Now $P(10)=11$ gives $(u-10)(v-10)mid 11$, showing that $u-10$ is a divisor of $11$. However, neither $1-10=-9$, not $3-10=-7$ is a divisor of $11$, a contradiction.






              share|cite|improve this answer














              If $u$ and $v$ are integer roots of $P$, then $P(x)=(x-u)(x-v)Q(x)$, where $Q$ is a polynomial with integer coefficients. From $P(2)=3$ we get $(u-2)(v-2)mid 3$, and then WLOG either $u-2=1$ or $u-2=-1$, implying $uin{1,3}$. Now $P(10)=11$ gives $(u-10)(v-10)mid 11$, showing that $u-10$ is a divisor of $11$. However, neither $1-10=-9$, not $3-10=-7$ is a divisor of $11$, a contradiction.







              share|cite|improve this answer














              share|cite|improve this answer



              share|cite|improve this answer








              edited Dec 30 '18 at 9:20

























              answered Dec 30 '18 at 9:14









              W-t-P

              92359




              92359








              • 1




                No idea why your answer has gotten so many fewer votes than mine! Your approach of considering both factors at once is quite slick.
                – Eric Wofsey
                Dec 30 '18 at 16:54






              • 2




                @EricWofsey: Don't worry, that's fine. Your answer provides some theoretical background and looks "more scientific". Besides, my empirical observation is that the number of upvotes is proportional to the present reputation of the person answering :-)
                – W-t-P
                Dec 30 '18 at 17:19














              • 1




                No idea why your answer has gotten so many fewer votes than mine! Your approach of considering both factors at once is quite slick.
                – Eric Wofsey
                Dec 30 '18 at 16:54






              • 2




                @EricWofsey: Don't worry, that's fine. Your answer provides some theoretical background and looks "more scientific". Besides, my empirical observation is that the number of upvotes is proportional to the present reputation of the person answering :-)
                – W-t-P
                Dec 30 '18 at 17:19








              1




              1




              No idea why your answer has gotten so many fewer votes than mine! Your approach of considering both factors at once is quite slick.
              – Eric Wofsey
              Dec 30 '18 at 16:54




              No idea why your answer has gotten so many fewer votes than mine! Your approach of considering both factors at once is quite slick.
              – Eric Wofsey
              Dec 30 '18 at 16:54




              2




              2




              @EricWofsey: Don't worry, that's fine. Your answer provides some theoretical background and looks "more scientific". Besides, my empirical observation is that the number of upvotes is proportional to the present reputation of the person answering :-)
              – W-t-P
              Dec 30 '18 at 17:19




              @EricWofsey: Don't worry, that's fine. Your answer provides some theoretical background and looks "more scientific". Besides, my empirical observation is that the number of upvotes is proportional to the present reputation of the person answering :-)
              – W-t-P
              Dec 30 '18 at 17:19











              1














              Key Idea $ $ (Kronecker) $ $ How polynomials can factor is constrained by how their values factor, $ $ e.g. as below, in some cases if $,P,$ takes a prime value then it has at most one integer root.



              Hint $ $ If $,P,$ has more roots than $,P(2),$ has prime factors then factoring $P$ & evaluating at $x!=!2$ $,Rightarrow,P(1)!=!0,$ or $P(3)!=!0.,$ But $P(1)!neq! 0,$ else $,10!-!1mid P(10)!-!P(1) = 11.,$ $P(3)!neq! 0,$ similarly.



              Theorem $ $ Suppose $P(x)$ is a polynomial with integer coefficients and $a$ is an integer with $,P(a)neq 0,$ and there exists an integer $b$ such that neither of $,b!-!apm 1$ divides $P(b).$



              $$begin{align} {rm Then} &P(a), text{has $, k, $ prime factors (counting multiplicity)}\
              Longrightarrow &P(x), text{ has $le! k,$ integer roots (counting multiplicty)}
              end{align}qquad $$



              Proof $ $ If not then $P$ has at least $,k+1,$ roots $,r_i,$ so iterating the Factor Theorem yields
              $$,P(x) = (x-r_0)cdots (x-r_k),q(x)qquad$$



              for a polynomial $,q(x),$ with integer coefficients. Evaluating above at $,x = a,$ yields



              $$,P(a) = (a-r_0)cdots (a-r_k),q(a)qquad$$



              If all $,a-r_ineq pm1,$ then they all have a prime factor yielding at least $k+1$ prime factors on the RHS, contra LHS $,P(a),$ has $,k,$ prime factors (prime factorizations are unique). So some $,a-r_j = pm1,$ so $,r_j = apm 1.,$ Evaluating at $, x = b,$ yields $,b-r_j = b-apm1,$ divides $, P(b),,$ contra hypothesis.






              share|cite|improve this answer




























                1














                Key Idea $ $ (Kronecker) $ $ How polynomials can factor is constrained by how their values factor, $ $ e.g. as below, in some cases if $,P,$ takes a prime value then it has at most one integer root.



                Hint $ $ If $,P,$ has more roots than $,P(2),$ has prime factors then factoring $P$ & evaluating at $x!=!2$ $,Rightarrow,P(1)!=!0,$ or $P(3)!=!0.,$ But $P(1)!neq! 0,$ else $,10!-!1mid P(10)!-!P(1) = 11.,$ $P(3)!neq! 0,$ similarly.



                Theorem $ $ Suppose $P(x)$ is a polynomial with integer coefficients and $a$ is an integer with $,P(a)neq 0,$ and there exists an integer $b$ such that neither of $,b!-!apm 1$ divides $P(b).$



                $$begin{align} {rm Then} &P(a), text{has $, k, $ prime factors (counting multiplicity)}\
                Longrightarrow &P(x), text{ has $le! k,$ integer roots (counting multiplicty)}
                end{align}qquad $$



                Proof $ $ If not then $P$ has at least $,k+1,$ roots $,r_i,$ so iterating the Factor Theorem yields
                $$,P(x) = (x-r_0)cdots (x-r_k),q(x)qquad$$



                for a polynomial $,q(x),$ with integer coefficients. Evaluating above at $,x = a,$ yields



                $$,P(a) = (a-r_0)cdots (a-r_k),q(a)qquad$$



                If all $,a-r_ineq pm1,$ then they all have a prime factor yielding at least $k+1$ prime factors on the RHS, contra LHS $,P(a),$ has $,k,$ prime factors (prime factorizations are unique). So some $,a-r_j = pm1,$ so $,r_j = apm 1.,$ Evaluating at $, x = b,$ yields $,b-r_j = b-apm1,$ divides $, P(b),,$ contra hypothesis.






                share|cite|improve this answer


























                  1












                  1








                  1






                  Key Idea $ $ (Kronecker) $ $ How polynomials can factor is constrained by how their values factor, $ $ e.g. as below, in some cases if $,P,$ takes a prime value then it has at most one integer root.



                  Hint $ $ If $,P,$ has more roots than $,P(2),$ has prime factors then factoring $P$ & evaluating at $x!=!2$ $,Rightarrow,P(1)!=!0,$ or $P(3)!=!0.,$ But $P(1)!neq! 0,$ else $,10!-!1mid P(10)!-!P(1) = 11.,$ $P(3)!neq! 0,$ similarly.



                  Theorem $ $ Suppose $P(x)$ is a polynomial with integer coefficients and $a$ is an integer with $,P(a)neq 0,$ and there exists an integer $b$ such that neither of $,b!-!apm 1$ divides $P(b).$



                  $$begin{align} {rm Then} &P(a), text{has $, k, $ prime factors (counting multiplicity)}\
                  Longrightarrow &P(x), text{ has $le! k,$ integer roots (counting multiplicty)}
                  end{align}qquad $$



                  Proof $ $ If not then $P$ has at least $,k+1,$ roots $,r_i,$ so iterating the Factor Theorem yields
                  $$,P(x) = (x-r_0)cdots (x-r_k),q(x)qquad$$



                  for a polynomial $,q(x),$ with integer coefficients. Evaluating above at $,x = a,$ yields



                  $$,P(a) = (a-r_0)cdots (a-r_k),q(a)qquad$$



                  If all $,a-r_ineq pm1,$ then they all have a prime factor yielding at least $k+1$ prime factors on the RHS, contra LHS $,P(a),$ has $,k,$ prime factors (prime factorizations are unique). So some $,a-r_j = pm1,$ so $,r_j = apm 1.,$ Evaluating at $, x = b,$ yields $,b-r_j = b-apm1,$ divides $, P(b),,$ contra hypothesis.






                  share|cite|improve this answer














                  Key Idea $ $ (Kronecker) $ $ How polynomials can factor is constrained by how their values factor, $ $ e.g. as below, in some cases if $,P,$ takes a prime value then it has at most one integer root.



                  Hint $ $ If $,P,$ has more roots than $,P(2),$ has prime factors then factoring $P$ & evaluating at $x!=!2$ $,Rightarrow,P(1)!=!0,$ or $P(3)!=!0.,$ But $P(1)!neq! 0,$ else $,10!-!1mid P(10)!-!P(1) = 11.,$ $P(3)!neq! 0,$ similarly.



                  Theorem $ $ Suppose $P(x)$ is a polynomial with integer coefficients and $a$ is an integer with $,P(a)neq 0,$ and there exists an integer $b$ such that neither of $,b!-!apm 1$ divides $P(b).$



                  $$begin{align} {rm Then} &P(a), text{has $, k, $ prime factors (counting multiplicity)}\
                  Longrightarrow &P(x), text{ has $le! k,$ integer roots (counting multiplicty)}
                  end{align}qquad $$



                  Proof $ $ If not then $P$ has at least $,k+1,$ roots $,r_i,$ so iterating the Factor Theorem yields
                  $$,P(x) = (x-r_0)cdots (x-r_k),q(x)qquad$$



                  for a polynomial $,q(x),$ with integer coefficients. Evaluating above at $,x = a,$ yields



                  $$,P(a) = (a-r_0)cdots (a-r_k),q(a)qquad$$



                  If all $,a-r_ineq pm1,$ then they all have a prime factor yielding at least $k+1$ prime factors on the RHS, contra LHS $,P(a),$ has $,k,$ prime factors (prime factorizations are unique). So some $,a-r_j = pm1,$ so $,r_j = apm 1.,$ Evaluating at $, x = b,$ yields $,b-r_j = b-apm1,$ divides $, P(b),,$ contra hypothesis.







                  share|cite|improve this answer














                  share|cite|improve this answer



                  share|cite|improve this answer








                  edited yesterday

























                  answered yesterday









                  Bill Dubuque

                  209k29190629




                  209k29190629















                      Popular posts from this blog

                      An IMO inspired problem

                      Management

                      Has there ever been an instance of an active nuclear power plant within or near a war zone?